Sie sind auf Seite 1von 5

STATS 200 (Stanford University, Summer 2015)

Solutions to Homework 4
DeGroot & Schervish X.Y .Z means Exercise Z at the end of Section X.Y in our text,
Probability and Statistics (Fourth Edition) by Morris H. DeGroot and Mark J. Schervish.
1. Let X be drawn from a discrete uniform distribution on {1, . . . , N }, where N 1 is an
= X. (We have
unknown positive integer. The maximum likelihood estimator of N is N
discussed this result before, so you do not need to show it.)
(as an estimator of N ).
(a) Find the bias, variance, and mean squared error of N
Note: The discrete uniform distribution on {1, . . . , N } has mean (N + 1)/2 and
variance (N 2 1)/12. You may use these facts without proof.
Solution: We have
) = EN (N
) N = EN (X) N = N + 1 N = ( N 1 ),
BiasN (N
2
2
2
) = Var(X) = N 1 ,
VarN (N
12
2

)
MSEN (N ) = [BiasN (N )] + VarN (N
N 1 2 N2 1 N 1
(N 1)(2N 1)
= [(
)] +
=
[3(N 1) + (N + 1)] =
,
2
12
12
6
using the facts provided.

that is an unbiased estimator of N .


(b) Find an estimator N
= 2X 1. Then EN (N
) = 2 EN (X)1 = N by the facts provided,
Solution: Let N
is an unbiased estimator of N .
so N

), and show that MSEN (N


) > MSEN (N
) for all N 2.
(c) Find MSEN (N
) = 0, we have
Solution: Since BiasN (N
2
) = VarN (N
) = VarN (2X 1) = 4 Var(X) = N 1 ,
MSEN (N
3

again using the facts provided. Then simply observe that


)
) = (N 1)(N + 1) = (N 1)(2N + 2) > (N 1)(2N 1) = MSEN (N
MSEN (N
3
6
6
for all N 2. (Note that if N = 1, then both estimators have an MSE of zero.)

Solutions to Homework 4

2. Let X Bin(n, ), where 0 < < 1 and is unknown. Let = 1/. Prove that no unbiased
estimator of exists.
Hint: Here, an estimator is fully specified by the value it takes for each x {0, . . . , n}.
Let be any arbitrary estimator of , and let tx be the value that takes when X = x.
when is specified in this way.
Then look at the form of E ()
Solution: Let be any estimator of . Using the hint, we can write E () as
n

n
E () = tx P (X = x) = tx ( ) x (1 )nx ,
x
x=0
x=0
which is some polynomial function of . For to be unbiased, this polynomial function
of must equal 1/ for all (0, 1). However, this is impossible. (If it is not immediately
clear why this is impossible, note that as 0, the polynomial function tends to whatever
finite value it takes at zero, whereas 1/ tends to .)

3. DeGroot & Schervish 8.8.2.


Note: The Geometric(p) distribution as given in Definition 5.5.2 of DeGroot & Schervish
has mean (1 p)/p. You may use this fact without proof.
Solution: The second derivative of the log-likelihood is
`X (p) =

1
X
1
X
2
[log p + X log(1 p)] =
(
)= 2
.
2
p
p p 1 p
p
(1 p)2

Then the Fisher information is


I(p) = Ep [`X (p)] = Ep [

1
X
1
1
1p
1

]
=
+
(
)
=
,
p2 (1 p)2
p2 (1 p)2 p
p2 (1 p)

noting that I(p) = I1 (p) since there is only one observation.

4. DeGroot & Schervish 8.8.14.


Solution: We begin by finding I1 (), the Fisher information per observation. The
second derivative of the log-likelihood of a single observation is
`X1 () =

()
[
log

log
()
+
(

1)
log
X

X
]
=
[log

+ log X1 ]
1
1
2

()
2

() () [ ()]

[()]

and thus
2

I1 () = E [`X1 ()] = E {

() () [ ()]
2

[()]

}=

() () [ ()]
2

[()]

The desired result then follows immediately by Theorem 7.2.4 of the lecture notes.

Solutions to Homework 4

5. Let X1 , . . . , Xn iid Bin(1, ), where 0 < < 1 and is unknown.


(a) Find the Fisher information I() for the sample.
Solution: We begin by finding I1 (), the Fisher information per observation.
The second derivative of the log-likelihood of a single observation is
`X1 () =

X1 1 X 1
X1 1 X1
2
[X1 log + (1 X1 ) log(1 )] = (

)= 2
,
2

(1 )2

and thus
I1 () = E [`X1 ()] = E [

X1 1 X1
1
1
1

]= +
=
.
2
2

(1 )
1 (1 )

Then the Fisher information for the entire sample is I() = n I1 () = n/[(1 )].
(b) We have shown before that the maximum likelihood estimator of is = n1 ni=1 Xi .
Does it agree
Use your answer to part (a) to state the asymptotic distribution of .
with the result obtained by using the central limit theorem?
Solution: By Theorem 7.2.4 of the lecture notes,

n(n ) D N [0, (1 )],


which agrees with the result obtained by using the central limit theorem.
6. Let X1 , . . . , Xn iid Pareto(k, ), where the Pareto(k, ) distribution has pdf

x+1
f (x) =

if x k,
if x < k.

Suppose that k > 0 is known and > 0 is unknown.


(a) Find the maximum likelihood estimator
of .
Solution: The derivative of the log-likelihood (i.e., the score function) is
`X ()

n
n

n
=
[n log + n log k ( + 1) log Xi ] = + n log k log Xi

i=1
i=1
n
n
Xi
= log( ),
i=1
k

which equals zero if and only if


1

1 n
Xi
= [ log( )] .
n i=1
k

Solutions to Homework 4

(Note that the right-hand side of the equation above is undefined if X1 = = Xn = k,


but we can ignore this possibility since it occurs with probability 0 for all > 0.)
Then since this is the only critical point and the log-likelihood clearly tends to
as 0 and as , it follows that this point is indeed the maximum. Thus,
Xi
1 n

n = [ log( )]
n i=1
k

is the maximum likelihood estimator of .


(b) Find the asymptotic distribution of
.

Solution: We begin by finding I1 (), the Fisher information per observation.


The second derivative of the log-likelihood of a single observation is
`X1 () =

1
1
( + log k log X1 ) = 2 ,

and thus
I1 () = E [`X1 ()] = E (

1
1
) = 2.
2

Then

n(
n ) D N (0, 2 )

by Theorem 7.2.4 of the lecture notes.

(c) Now suppose instead that k > 0 is unknown and > 0 is known. Compute Ek [`X (k)],
and explain why your answer does not contradict Lemma 7.2.1 from the lecture notes.
Solution: The score function is now
`X (k) =

n
n

[n log + n log k ( + 1) log Xi ] =


,
k
k
i=1

and thus
Ek [`X (k)] = Ek (

n
n
)=
0.
k
k

At first glance, it may appear that this result contradicts Lemma 7.2.1 of the lecture
notes, which states that the expectation of the score is zero. However, there is no
contradiction since one of the regularity conditions of Section 7.4 is now violated,
which means that Lemma 7.2.1 does not apply. Specifically, the set
X = {x R `X1 (k) > 0} = {x R x k} = [k, )
depends on the unknown parameter k, which is not permitted by the regularity
conditions of Section 7.4. (Note that there would be no violation here if instead k
were known, as was the case in the previous question.)

Solutions to Homework 4

7. Let X1 , . . . , Xn be iid continuous random variables with a pdf f (x) that is symmetric
about , where R is unknown. Suppose that Var (X1 ) = 2 < is known, which
implies that E (X1 ) = . Then is both the true mean and true median of the pdf f (x),
so it seems plausible that both the sample mean X n and the sample median, which we
will call Mn , could be good estimators of .
(a) Use the central limit theorem to state the asymptotic distribution of the sample
mean X n .

Solution: n(X n ) D N (0, 2 ).


Suppose f (), the value of the pdf at the true mean (and median), satisfies f () > 0.
Then it can be shown that the asymptotic distribution of the sample median Mn is

n(Mn ) D N (0,

2 ).

4[f ()]

(You do not need to show this.)


(b) Suppose f (x) is the pdf of a N (, 2 ) distribution, where 2 > 0 is known. Find the
asymptotic relative efficiency of the sample median compared to the sample mean,
and use it to state which estimator performs better asymptotically.
Solution: ARE(Mn , X n ) = 4[f ()]2 /(1/ 2 ) = 4(2 2 )1 2 = 2/ 0.64, so the
sample mean performs better asymptotically.

(c) Suppose f (x) = 12 exp(x ), where > 0 is known. Find the asymptotic
relative efficiency of the sample median compared to the sample mean, and use it to
state which estimator performs better asymptotically.
Note: Under this pdf, Var (X1 ) = 2/2 . You may use this fact without proof.
Solution: ARE(Mn , X n ) = 4[f ()]2 /(1/ 2 ) = 4( 21 )2 /( 21 2 ) = 2, so the sample
median performs better asymptotically.

Das könnte Ihnen auch gefallen